LSAT and Law School Admissions Forum

Get expert LSAT preparation and law school admissions advice from PowerScore Test Preparation.

 Administrator
PowerScore Staff
  • PowerScore Staff
  • Posts: 8916
  • Joined: Feb 02, 2011
|
#73113
Complete Question Explanation

Strengthen-PR. The correct answer choice is (D).

Our author argues that the government should not help local residents with rebuilding hiking trails that were damaged by a landslide, on the grounds that more landslides could occur later that would cause serious injury. The stem asks us for a Principle that would "help to justify" that reasoning. Note that the presence of the qualifier "help" means that this is not a Justify the Conclusion question, but merely a Strengthen question, where a lesser degree of help would be acceptable.

To help this argument, the correct answer will provide a rule that connects the evidence (risk of serious injury) more directly to the conclusion (the government should not help). Strengthen answers typically provide a link between the premises and the conclusion, and prephrases will often take the form of a conditional statement, such as, in this case, "if an action might lead to serious injury, the government should not take that action."

Answer choice (A): This rule tells us what residents should not be allowed to do, rather than what the government should not do, and it therefore adds nothing to the claim that the government should not help in this case.

Answer choice (B): An opposite answer, this rule, if valid, weakens the argument, because the local residents are strongly committed to the project. If their determination is what determines the governments actions, this would suggest that the government should help to rebuild, the exact opposite of the conclusion we are looking to strengthen.

Answer choice (C): Since our local residents are strongly committed to rebuilding the hiking trails, this answer does nothing to support the conclusion that they should not help in this case.

Answer choice (D): This is the correct answer choice. Here we find our prephrase, and a solid link between the premises about the possibility of serious injury and the conclusion that the government should not help to rebuild. If this rule was added to the stimulus as a premise, the conclusion would look much more certain.

Answer choice (E): As there is no information in the stimulus about an extensive history of the area (just one recent landslide), and because this answer tells us nothing about what governments should do, it does nothing to help support the conclusion that the government should not, in this case, help to rebuild the hiking trails.

Get the most out of your LSAT Prep Plus subscription.

Analyze and track your performance with our Testing and Analytics Package.